A cylindrical tank whose diameter is 1.4 metres and height 80 cm is initially empty. Water whose volume is 492.8 litres is poured into the tank. Determine the fraction of the tank filled with water. (4 mark

Answers

Answer 1

Answer:

The fraction filled with water is 7/40

Step-by-step explanation:

Okay.

Here we start by calculating the volume of the cylindrical tank.

Mathematically, that would be;

V = π * r^2 * h

From the question

r = 80 cm = 80/100 = 0.8 meters

h = 1.4 meters

π = 22/7

Plugging these values into the volume equation, we have;

V = 22/7 * 0.8 * 0.8 * 1.4 = 2.816 m^3

But mathematically;

1 m^3 = 1000 liters

So 2.816 m^3 = 2.816 * 1000 = 2816 liters

So the fraction filled with water will be;

492.8/2816 = 0.175 = 175/1000 = 7/40


Related Questions

I am visiting my friend Janette in Bristol. My journey takes a total time of 1 hour 26 minutes. I travel by train for 34 minutes, then walk at a rate of 1/2 mile per 10 minutes. How many miles do I walk for?

Answers

Answer:

2.6 miles

Step-by-step explanation:

1 hour 26 minutes= 86 minutes

86-34=52 total walk time

52 minutes= 5*1/2 miles

=2.5 miles walked

and 2 minutes.

so we need to find 1/5 of 1/2

(1/2)/5=0.1 mile

2.5+0.1=2.6 miles

is 5.676677666777 a rational number​

Answers

Answer:Yes, because all integers have decimals. No, because integers do not have decimals. No, because integers cannot be negative. Jeremy says that 5.676677666777... is a rational number because it is a decimal that goes on forever with a pattern.

Step-by-step explanation:

solve this equation -2x+9=-5x-15

Answers

Answer:

x = -8

I hope this helps!

The answer would be x=-8

Reduce any fractions to lowest terms. Don't round your answer, and don't use mixed fractions. 4x+4\leq9x+84x+4≤9x+8

Answers

Answer:

  x ≥ -4/5

Step-by-step explanation:

Maybe you want to solve ...

  4x+4 ≤ 9x +8

  0 ≤ 5x +4 . . . . . subtract 4x+4

  0 ≤ x +4/5 . . . . . divide by 5

  -4/5 ≤ x . . . . . . . subtract 4/5

Answer:

x ≥−4/5

Step-by-step explanation:

This rectangular wall is to be painted. Paint is sold in tins. How much does it cost to paint the wall?

Answers

Answer:

£23.96

Step-by-step explanation:

Area to be painted:

3.6 m * 8.3 m = 29.88 m^2

The area to be painted is 29.88 m^2.

A tin of paint covers 8 m^2. We divide to find the number of tins needed.

29.88/8 = 3.735

Since full tins must be bought, the smallest number of tins needed is 4.

Now we find the price of 4 tins. 1 tin costs £5.99, so 4 tins cost:

4 * £5.99 = £23.96

If you make $3.80 an hour plus tips, what is your paycheck for the week if you worked 40 hours and made $250.00 dollars in tips?

Answers

Answer:

$402

Step-by-step explanation:

Hello!

If you made 3.80 an hour and worked 40 we can multiply these to find the total amount you earned.

3.80 * 40 = 152

You also made 250 in tips so we add that to the total

152+250 = 402

The answer is $402

Hope this helps!

I would make $402.00 by the end of the week

Determine the equation of the graph and select the correct answer below.
(1, 1-3)
Courtesy of Texas Instruments

Answers

Answer:

  y = (x -1)² -3

Step-by-step explanation:

A quadratic with a vertex at (h, k) will have an equation of the form ...

 y = a(x -h)² +k

You have (h, k) = (1, -3), and a vertical scale factor* of 1. So, the equation of the graphed curve is ...

  y = (x -1)² -3

_____

* One way to determine the value of "a" in the form shown is to look at the vertical difference between the vertex and the points 1 unit right or left of the vertex. Here, those points are 1 unit above the vertex, so the vertical scale factor "a" is 1.

If 4SINB=3SIN(2A+B) :
Prove that:7COT(A+B)=COTA

Answers

Answer:

Step-by-step explanation:

Given the expression 4sinB = 3sin(2A+B), we are to show that the expression 7cot(A+B) = cotA

Starting with the expression

4sinB= 3sin(2A+B)

Let us re write angle B = (A + B) - A

and 2A + B = (A + B) + A

Substituting the derived expression back into the original expression ww will have;

4Sin{(A + B) - A } = 3Sin{(A + B)+ A}

From trigonometry identity;

Sin(D+E) = SinDcosE + CosDSinE

Sin(D-E) = SinDcosE - CosDSinE

Applying this in the expression above;

4{Sin(A+B)CosA - Cos(A+B)SinA} = 3{Sin(A+B)CosA + Cos(A+B)sinA}

Open the bracket

4Sin(A+B)CosA - 4Cos(A+B)SinA = 3Sin(A+B)CosA + 3Cos(A+B)sinA

Collecting like terms

4Sin(A+B)CosA - 3Sin(A+B)cosA = 3Cos(A+B)sinA + 4Cos(A+B)sinA

Sin(A+B)CosA = 7Cos(A+B)sinA

Divide both sides by sinA

Sin(A+B)CosA/sinA= 7Cos(A+B)sinA/sinA

Since cosA/sinA = cotA, the expression becomes;

Sin(A+B)cotA = 7Cos(A+B)

Finally, divide both sides of the resulting equation by sin(A+B)

Sin(A+B)cotA/sin(A+B) = 7Cos(A+B)/sin(A+B)

CotA = 7cot(A+B) Proved!

how many are 6 raised to 4 ???​

Answers

Answer:

[tex]\large \boxed{1296}[/tex]

Step-by-step explanation:

6 raised to 4 indicates that the base 6 has an exponent or power of 4.

[tex]6^4[/tex]

6 is multiplied by itself 4 times.

[tex]6 \times 6 \times 6 \times 6[/tex]

[tex]=1296[/tex]

If x^2 -8x=48 and x<0, what is the value of x+10?

Answers

Answer:

6

Step-by-step explanation:

To calculate x+10, we first need to find x. To do this, we can use the first equation.

We are given the equation:

[tex]x^2-8x=48[/tex]

To solve for x, turn one side of the equation into 0 and solve. Therefore:

[tex]x^2-8x=48\\x^2-8x-48=0\\(x-12)(x+4)=0\\x=-4, 12[/tex]

So, the possible values for x are -4 and 12.

However, we are also told that x<0. In other words, x must be negative. Thus, we can remove 12. That leaves us with: x=-4.

So:

[tex]x+10\\(-4)+10\\=6[/tex]

x^2 -8x - 48 = 0
(x-12)(x+4) = 0
x=12, x=-4
Since x<0, x must be -4
Therefore, -4+10=6

How can I divide decimals and fin the correct quotient and remainder.?

Answers

Answer:

Add a zero to the remainder and a decimal point in the quotient. Then we can continue to divide decimals. We divide 64 by 5 and obtain 12 as a quotient and 4 as a remainder. Since the remainder is not zero, we can continue to get a decimal answer by adding a decimal point in the quotient and a zero to the remainder

Step-by-step explanation:

Pls help, I don’t know how to fo

Answers

frustum of a cone is: = pi * l(R + r)

(l) = slant height of the frustum.

from 2929.645714 - 506.1257143

= 2423.52

= 2423.5cm

Answer:

from 2929.645714 - 506.1257143

= 2423.52

= 2423.5cm

Please answer this question now in two minutes

Answers

Answer:

m∠C = 102°

Step-by-step explanation:

This diagram is a Quadrilateral inscribed in a circle

The first step is to determine what m∠B

is

The sum of opposite angles in an inscribed quadrilateral is equal to 180°

m∠D + m∠B = 180°

m∠B = 180° - m∠D

m∠B = 180° - 80°

m∠B = 100°

Second step is we proceed to determine the exterior angles of the circle

m∠ADC = 2 × m∠B

m∠ADC = 2 × 100°

m∠ADC = 200°

m∠ADC = m∠CD + m∠AD

m∠AD = m∠ADC - m∠CD

m∠AD = 200° - 116°

m∠AD = 84°

The third step is to determine m∠BAD

m∠BAD = m∠AD + m∠AB

m∠BAD = 84° + 120°

m∠BAD = 204°

The final step Is to determine what m∠C is

It is important to note that:

m∠BAD is Opposite m∠C

Hence

m∠C = 1/2 × m∠BAD

m∠C = 1/2 × 204

m∠C = 102°

Can someone help me with this please it’s algebra 2

Answers

Answer:

7 8 9

Step-by-step explanation:

The height of a building model is 2% of its actual height. If the building
model is 3 feet tall, how tall is the actual building?

Answers

Answer:

x = 150 feets

Step-by-step explanation:

Given that,

The height of a building model is 2% of its actual height.

The building model is 3 feet tall, h = 3 feet

We need to find the height of the actual building. Let it is x.

According to question,

h = 2% of x

We have, h = 3 feet

So,

[tex]x=\dfrac{h}{2\%}\\\\x=\dfrac{3}{2/100}\\\\x=150\ \text{feet}[/tex]

So, the actual height of the building is 150 feets.

the perimeter of square is 76 cm find are of square ​

Answers

Answer:

Given the information above, the area of the square is 361 cm²

Step-by-step explanation:

A square is a shape with four equal sides. So, in order to find the area of the square, we must find the length of each individual side. We can do this by dividing the perimeter by 4 because a square has 4 equal sides meaning they have the same lengths.

The perimeter of the square is 76. So, let's divide 76 by 4.

76 ÷ 4 = 19

So, the lengths of each sides in the square is 19cm.

In order to find the area, we must multiply the length and the width together. Since a square has equal sides, then we will multiply 19 by 19 to get the area.

19 × 19 = 361

So, the area of the square is 361 cm²

Answer:

361 cm^2

Step-by-step explanation:

The area of a square can be found by squaring the side length.

[tex]A=s^2[/tex]

A square has four equal sides. The perimeter is the sum of all four sides added together. Therefore, we can find one side length by dividing the perimeter by 4.

[tex]s=\frac{p}{4}[/tex]

The perimeter is 76 centimeters.

[tex]s=\frac{76 cm}{4}[/tex]

Divide 76 by 4.

[tex]s=19 cm[/tex]

The side length is 19 centimeters.

Now we know the side length and can plug it into the area formula.

[tex]A=s^2\\s=19cm[/tex]

[tex]A= (19 cm)^2[/tex]

Evaluate the exponent.

(19cm)^2= 19 cm* 19cm=361 cm^2

[tex]A= 361 cm^2[/tex]

The area of the square is 361 square centimeters.

The graph of f(x) = StartRoot x EndRoot is reflected over the y-axis. Use the graphing calculator to graph this reflection. Which list contains three points that lie on the graph of the reflection? (–81, 9), (–36, 6), (–1, 1) (1, –1), (16, –4), (36, –6) (–49, 7), (–18, 9), (–1, 1) (1, –1), (4, –16), (5, –25)

Answers

Answer:

(–81, 9), (–36, 6), (–1, 1) are the correct three points.

Step-by-step explanation:

Given the function:

[tex]f(x) =\sqrt x[/tex]

Please refer to the attached image.

The green line shows the graph of actual function.

It is reflected over y axis.

The reflected graph is shown in black color in attached image.

When reflected over y axis, the sign of variable [tex]x[/tex] changes from Positive to Negative.

So, the resultant function becomes:

[tex]f(x)=\sqrt{-x}[/tex]

i.e. we will have to give the values of x as negative now.

so, the options in which value of x is negative are the possible answers only.

The possible answers are:

(–81, 9), (–36, 6), (–1, 1) and

(–49, 7), (–18, 9), (–1, 1)

Now, we will check the square root function condition.

In the 2nd option, (–18, 9) does not satisfy the condition.

So, the correct answer is:

(–81, 9), (–36, 6), (–1, 1)

Answer:

A on E2020

Step-by-step explanation:

:)

PLEASE HELP QUICK, WILL MARK BRAINLIEST!
Solve for x: −6 < x − 1 < 9

5 < x < 10
−5 < x < 10
−5 > x > 10
5 > x > −10

Answers

Answer:

−5 < x < 10

Step-by-step explanation:

−6 < x − 1 < 9

Add 1 to all sides

−6+1 < x − 1+1 < 9+1

−5 < x < 10

Answer:

B

Step-by-step explanation:

Add one to everything

-5 < x < 10

Best of Luck!

I need help factoring this question, Factor 4(20) + 84.

Answers

Answer:

164

Step-by-step explanation:

B for brackets

O for of

D for division

M for multiplication

A for addition

S for subtraction

You first start with the brackets (20) and multiply with 4 which is equal to 80 and then add it to 84 which makes 164

I hope this helps

1-Determine a solução dos sistemas abaixo pelo método de adição: a) {x + y = 5 {2x- y=9 b) {3x - y = 10 {x + y =18 Prfvr gente

Answers

a)

X + Y = 5

2X - Y = 9      

X + 2X + Y - Y = 5 + 9

3X = 14

X = 14/3

Para Y, basta substituir o valor de X em qualquer uma das 2 equacoes - arbitrariamente. Escolhendo a primeira:

X+ Y = 5

14/3 + Y = 5

Y = 5 - 14/3

Y = 1/3

.........................

b)

3X - Y = 10

X + Y = 18        

3X + X - Y + Y = 10 + 18

4X = 28

X = 7

Para Y, basta substituir o valor de X em qualquer uma das 2 equacoes - arbitrariamente. Escolhendo a segunda:

X + Y = 18

7 + Y = 18

Y = 18 - 7

Y = 11

A zoo train ride costs $4 per adult and $1 per child. On a certain day, the total number of adults (a) and children (c) who took the ride was 27, and the total money collected was $60. What was the number of children and the number of adults who took the train ride that day, and which pair of equations can be solved to find the numbers? 1) 11 children and 16 adults Equation 1: a + c = 27 Equation 2: 4a + c = 60 2) 16 children and 11 adults Equation 1: a + c = 27 Equation 2: 4a + c = 60 3) 11 children and 16 adults Equation 1: a + c = 27 Equation 2: 4a − c = 60 4) 16 children and 11 adults Equation 1: a + c = 27 Equation 2: 4a − c = 60

Answers

Answer:

11 adults and 16 children

Step-by-step explanation:

a + c = 27 and 4a + c = 60

3a = 60 - 27 = 33

a= 11  

so c = 16

A combination lock uses three numbers between 1 and 46 with​ repetition, and they must be selected in the correct sequence. Is the name of​ "combination lock"​ appropriate? Why or why​ not? Choose the correct answer below. A. ​No, because the multiplication counting rule would be used to determine the total number of combinations. B. ​Yes, because the combinations rule would be used to determine the total number of combinations. C. ​No, because factorials would be used to determine the total number of combinations. D. ​No, because the permutations rule would be used to determine the total number of combinations.

Answers

The correct answer is D. ​No because the permutations rule would be used to determine the total number of combinations.

Explanation:

The difference between a combination and a permutation is that in permutations the order is considered. This applies to the numbers in a lock because these need to be in order. Therefore, to analyze the permutations in a lock, the rule for permutations should be used. This includes the general formula P (n,r) =[tex]\frac{n!}{(n-r) !}[/tex]; in this, n is the number of objects and r refers to the objects used in a permutation. Thus, the term "combination" is inappropriate because this is a permutation, and the permutation rule should be used.


Write the equation of the line which passes
through the points (4,2) and (-3, 1)

Answers

Answer:

y = 1/3x + 4/7

Step-by-step explanation:

Slope Formula: [tex]m=\frac{y_2-y_1}{x_2-x_1}[/tex]

Slope-Intercept Formula: y = mx + b

Step 1: Find slope m

m = (1 - 2)/(-3 - 4)

m = -1/-7

m = 1/7

y = 1/7x + b

Step 2: Find y-intercept b

1 = 1/7(3) + b

1 = 3/7 + b

b = 4/7

Step 3: Write linear equation

y = 1/3x + 4/7

Solve. 4x−y−2z=−8 −2x+4z=−4 x+2y=6 Enter your answer, in the form (x,y,z), in the boxes in simplest terms. x= y= z=

Answers

Answer:

(-2, 4, 2)

Where x = -2, y = 4, and z = 2.

Step-by-step explanation:

We are given the system of three equations:

[tex]\displaystyle \left\{ \begin{array}{l} 4x -y -2z = -8 \\ -2x + 4z = -4 \\ x + 2y = 6 \end{array}[/tex]

And we want to find the value of each variable.

Note that both the second and third equations have an x.

Therefore, we can isolate the variables for the second and third equation and then substitute them into the first equation to make the first equation all one variable.

Solve the second equation for z:

[tex]\displaystyle \begin{aligned} -2x+4z&=-4 \\ x - 2 &= 2z \\ z&= \frac{x-2}{2}\end{aligned}[/tex]

Likewise, solve the third equation for y:

[tex]\displaystyle \begin{aligned} x+2y &= 6\\ 2y &= 6-x \\ y &= \frac{6-x}{2} \end{aligned}[/tex]

Substitute the above equations into the first:

[tex]\displaystyle 4x - \left(\frac{6-x}{2}\right) - 2\left(\frac{x-2}{2}\right)=-8[/tex]

And solve for x:

[tex]\displaystyle \begin{aligned} 4x+\left(\frac{x-6}{2}\right)+(2-x) &= -8 \\ \\ 8x +(x-6) +(4-2x) &= -16 \\ \\ 7x-2 &= -16 \\ \\ 7x &= -14 \\ \\ x &= -2\end{aligned}[/tex]

Hence, x = -2.

Find z and y using their respective equations:

Second equation:

[tex]\displaystyle \begin{aligned} z&=\frac{x-2}{2} \\ &= \frac{(-2)-2}{2} \\ &= \frac{-4}{2} \\ &= -2\end{aligned}[/tex]

Third equation:

[tex]\displaystyle \begin{aligned} y &= \frac{6-x}{2}\\ &= \frac{6-(-2)}{2}\\ &= \frac{8}{2}\\ &=4\end{aligned}[/tex]

In conclusion, the solution is (-2, 4, -2)

Answer:

x = -2

y =4

z=-2

Step-by-step explanation:

4x−y−2z=−8

−2x+4z=−4

x+2y=6

Solve the second equation for x

x = 6 -2y

Substitute into the first two equations

4x−y−2z=−8

4(6-2y) -y -2 = 8  

24 -8y-y -2z = 8

-9y -2z = -32

−2(6-2y)+4z=−4

-12 +4y +4z = -4

4y+4z = 8

Divide by 4

y+z = 2

z =2-y

Substitute this into -9y -2z = -32

-9y -2(2-y) = -32

-9y -4 +2y = -32

-7y -4 = -32

-7y =-28

y =4

Now find z

z = 2-y

z = 2-4

z = -2

Now find x

x = 6 -2y

x = 6 -2(4)

x =6-8

x = -2

Of the 40 specimens of bacteria in a dish, 3 specimens have a certain trait. If 5 specimens are to be selected from the dish at random and without replacement, which of the following represents the probability that only 1 of the 5 specimens selected will have the trait?1) (5/1)/(40/3)
2) (5/1)/(40/5)
3) (40/3)/(40/5)
4) (3/1)(37/4)/(40/3)
5) (3/1)(37/4)/(40/5)

Answers

Answer:

[tex]\frac{(^3_1)*(^{37}_4)}{(^{40}_5)}[/tex]

Step-by-step explanation:

The total number of ways in which 5 specimens can be selected from the  dish at random is given as C(40, 5).

Since only one of the five specimens would have the trait, the number of ways of selecting the one specimen out of the 3 specimens with the trait is C(3, 1).

3 specimens have the trait therefore 37 specimens (40 - 3) do not have the trait. The number of ways in which the remaining 4 specimens out of the 5 spemimens that do not have the trait is C(37, 4).

Therefore, the probability that only 1 of the 5 specimens selected will have the trait = [tex]\frac{C(3,1)*C(37,4)}{C(40,5)} =\frac{(^3_1)*(^{37}_4)}{(^{40}_5)}[/tex]

Two birds sit at the top of two different trees. The distance between the first bird and a birdwatcher on the ground is 32 feet. The distance between the birdwatcher and the second bird is 45 feet. A triangle is created from point Bird Watcher, point First Bird, and point Second Bird. Angle First Bird is a right angle, and angle Second Bird measures x degrees. What is the angle measure, or angle of depression, between this bird and the birdwatcher? Round your answer to the nearest tenth. 35.4° 44.7° 45.3° 54.6°

Answers

Answer:

Step-by-step explanation:

When you draw out that picture (and very good description, btw!) basically what you have is a right triangle that has a base of 32 and a hypotenuse of 45. The right angle is one of the base angles and x is the vertex angle. We need to find the vertex angle before we can find the angle of depression from the second bird to the watcher. The side of length 32 is opposite the angle x, and 45 is the hypotenuse, so the trig ratio we need is the only one that directly relates side opposite to hypotenuse, which is the sin ratio:

[tex]sin(x)=\frac{32}{45}[/tex] and

sin(x) = .711111111

Go to your calculator and hit the 2nd button then the sin button and on your screen you will see:

[tex]sin^{-1}([/tex]  

and after that open parenthesis enter in your decimal .711111111 and hit equals. You should get an angle of 45.325. That's angle x. But that's not the angle of depression. The angle of depression is the one complementary to angle x.

Angle of depression = 90 - angle x and

Angle of depression = 90 - 45.325 so

Angle of depression = 44.67 or 44.7 degrees.

Answer:

Its 45.3!!!

Step-by-step explanation:

determine the image of the point p[-3,10) under the translation [5,-7]

Answers

[tex](-3+5,10-7)=(2,3)[/tex]



A ship drops its anchor into the water and creates a circular ripple. The radius of the ripple increases at

a rate of 50 cm/s. If the origin is used as the location where the anchor was dropped into the water.

Find the equation for the circle 12 seconds after the anchor is dropped


Please write all the steps it’s for my summer school test and I need it done quick as possible thanks.

Answers

Answer:

The equation for the circle 12 seconds after the anchor is dropped is x^2 + y^2 = 360,000

Step-by-step explanation:

To find the equation for the circle 12 seconds when the radius of the ripple increases at a rate of 50 cm/s, the circle radius will be;

50 * 12 = 600 cm

Then place the equation inform of Pythagoras equation which is;

x^2 + y^2 = r^2

Where r is the radius

x^2 + y^2 = 600^2

x^2 + y^2 = 360,000

Then, the equation for the circle 12 seconds after the anchor is dropped is x^2 + y^2 = 360,000

Use the motion map to answer the question.
Which scenario could be represented by the motion
map?

O A car speeds up to merge onto the freeway and
then continues at a constant velocity
O A car speeds up to pass a truck, then slows down
to a constant velocity.
O A car slows to stop at a stop sign. Once traffic is
clear, the car speeds up.
O A car slows to makes a U-turn, then continues in
the opposite direction.

Answers

Answer:

A car slows to stop at a stop sign. Once traffic is clear, the car speeds up.

Step-by-step explanation:

Answer:

C.) A car slows to stop at a stop sign. Once traffic is clear, the car speeds up.

Step-by-step explanation:

A researcher performs a hypothesis test to test the claim that for a particular manufacturer, the mean weight of cereal in its 18 ounce boxes is less than 18 ounces. He uses the following hypotheses: H 0: μ = 18 vs H A: μ < 18 and finds a P-value of 0.01. Draw a conclusion about the cereal box weight at a significance level of 0.05.

Answers

Answer:

We conclude that the mean weight of cereal in its 18-ounce boxes is less than 18 ounces.

Step-by-step explanation:

We are given that a researcher performs a hypothesis test to test the claim that for a particular manufacturer, the mean weight of cereal in its 18-ounce boxes is less than 18 ounces.

Let [tex]\mu[/tex] = mean weight of cereal in its 18-ounce boxes.

So, Null Hypothesis, [tex]H_0[/tex] : [tex]\mu[/tex] = 18       {mean that the mean weight of cereal in its 18-ounce boxes is equal to 18 ounces}

Alternate Hypothesis, [tex]H_A[/tex] : [tex]\mu[/tex] < 18      {mean that the mean weight of cereal in its 18-ounce boxes is less than 18 ounces}

Also, it is given that the P-value is 0.01 and the level of significance is 0.05.

The decision rule based on the P-value is given by;

If the P-value of our test statistics is less than the level of significance, then we have sufficient evidence to reject our null hypothesis as our test statistics will fall in the rejection region.If the P-value of our test statistics is more than the level of significance, then we have insufficient evidence to reject our null hypothesis as our test statistics will not fall in the rejection region.

Here, clearly our P-value is less than the level of significance as 0.01 < 0.05, so we have sufficient evidence to reject our null hypothesis as our test statistics will fall in the rejection region.

Therefore, we conclude that the mean weight of cereal in its 18-ounce boxes is less than 18 ounces.

Other Questions
12. Consider the function (x) = x^4 x^3 + 2x^2 2x. How many real roots does it have? options: A) 2 B) 1 C) 3 D) 4 Which, if any, pair of sides are parallel? AB II DC and AD II BC Cannot be determined AB II DC only AD II BC only The lengths of two sides of an isosceles triangle are 5 and 9. The length of the third side could be Research an organization that makes people their primary focus and another organization that makes productivity and efficiency their primary focus. Compare, contrast, and discuss the control techniques and measurements for each organization. When the Federal Reserve buys long term MBS and Treasury securities from banks and announces its intention to keep buying these assets in large quantities for a long time the effect on commercial banks is to increase the value of fixed income securities that are not sold and at the same time to lower the interest spread between new loans originated and the cost of financing these loans. True False What did both the Aztecs and the Incas have in common Given: AB tangent at D, AD = OD = 4 Find: Area of the shaded region Assuming all blood types are equally common, people with what blood types would have the best and worst odds of finding a donor? Which statement is most in line with conservatives' perception of poverty? A. Poverty results from societal failures. B. Means-tested anti-poverty programs can encourage laziness among the poor. C. Means-tested welfare programs help people navigate difficult circumstances. D. Poverty is mostly a rural problem, especially in Appalachia. Solve the following system of equations.2x + y = 3x = 2y-1ANSWER: ______plz help me Martin and Sons (M and S) currently is an all equity firm with 64,000 shares of stock outstanding at a market price of $25 a share. The company's earnings before interest and taxes are $86,000. M and S has decided to add leverage to their financial operations by issuing $480,000 of debt with a 7% percent interest rate. This $480,000 will be used to repurchase shares of stock. You own 2,700 shares of M and S stock. You also loan out funds at a 7% percent rate of interest. How many of your shares of stock in M and S must you sell to offset the leverage that the firm is assuming Write the polar form of a complex number in standard form for [tex]8[cos(\frac{\pi}{2}) + isin(\frac{\pi}{2})][/tex] Curry's actions in the Spanish American Wara. did not amount to anything, because he never got to Cuba.b. were seen by Roosevelt as criminal.earned him a lifetime of friendship and loyalty with Roosevelt.d. were seen by New Mexicans as loyal.C.Its 1. Answer the following question with a complete sentence. Begin your answer with either s or no. Te gustan los exmenes de espaol? 2. Answer the following question with a complete sentence using the cue in parentheses. Qu hiciste ayer? (trabajar) 3. Answer the following question with a complete sentence using the cue in parentheses. Adnde fuiste el fin de semana pasado? (ir al cine) 4. Answer the following question with a complete sentence using the cue in parentheses. A qu hora te acuestas normalmente? (11:00 de la noche) 5. Answer the following question with a complete sentence using the cue in parentheses. A qu hora empieza la clase de matemticas? (10:00 de la maana) 6. Answer the following question with a complete sentence using the cue in parentheses. Qu sirvi mam para la cena? (hamburguesas) 7. Answer the following question with a complete sentence using the cue in parentheses. Cmo eres? (divertido/a y cmico/a) 8. Use the prompt in parentheses to write a complete sentence that answers the following question. Qu hiciste durante el fin de semana? (jugar baloncesto) 9. Use the prompt in parentheses to write a complete sentence that answers the following question. Hace cunto tiempo que Marta estudia espaol? (2 aos) 10. Answer the following question in a complete sentence that starts with yes or no. Quieres viajar a Colombia? Which of the following approaches should the Fed use if it experiences large lags and mistakes in monetary policy?a. Discretionary policy b. An eclectic approach c. Fixed rules d. Fiscal policy Which list describes the aspects of health that should be in balance to achieve total wellness? OA. nutritional, physical, and mental OB. physical, social, and emotional O c. intellectual, short term, and long term O D. cultural, technological, and analytical ABC is an eqilateral triangle with sides 10cm find the length of the prependicular from A to BC which of the following molecules would you expect to be optically active 1. CCl2F2 2. 2-methyl butane 3. butan-2-ol Midwest Fabricators Inc. is considering an investment in equipment that will replace direct labor. The equipment has a cost of $85,000 with a $7,000 residual value and a ten-year life. The equipment will replace one employee who has an average wage of $20,210 per year. In addition, the equipment will have operating and energy costs of $4,130 per year. Determine the average rate of return on the equipment, giving effect to straight-line depreciation on the investment. If required, round to the nearest whole percent. % please help me i need help